Final Review (Updated)
Final Review (Updated)
Below is a list of problems for the final exam. Do check out the problems worked out in the examples in the lecture
notes and discussed in class.
−1 −1 −1 −1 −1 −1
⇒ ( 𝐴𝐵) −1 = 𝐵−1 𝐴−1 .
( 𝐴𝐵)(𝐵 𝐴 ) = 𝐴(𝐵𝐵 ) 𝐴 = 𝐴(𝐼) 𝐴 = 𝐴𝐴 = 𝐼.
The above shows that ( 𝐴𝑇 ) −1 = ( 𝐴−1 )𝑇 . Since 𝐴𝑇 = 𝐴, this shows that 𝐴−1 = ( 𝐴−1 )𝑇 .
Thus, 𝐴−1 is symmetric.
(20) Let 𝐴, 𝐵 ∈ 𝑀𝑛 (R) such that 𝐴2 = 𝐼 and 𝐵2 = 𝐵. Show that (2𝐵 − 𝐼) 2 = 𝐼 and ( 𝐴 + 𝐼) 2 =
2( 𝐴 + 𝐼).
Solution. We have
(2𝐵 − 𝐼) 2 = (2𝐵 − 𝐼)(2𝐵 − 𝐼) = 4𝐵2 − 2𝐵 − 2𝐵 + 𝐼 = 4𝐵 − 4𝐵 + 𝐼 = 𝐼, (using 𝐵2 = 𝐵)
and
( 𝐴 + 𝐼) 2 = ( 𝐴 + 𝐼)( 𝐴 + 𝐼) = 𝐴2 + 𝐴 + 𝐴 + 𝐼 = 𝐼 + 2𝐴 + 𝐼 = 2( 𝐴 + 𝐼), (using 𝐴2 = 𝐼).
(21) Let 𝑥 represent a student in this class, and 𝑃(𝑥) and 𝑄(𝑥) be the following predicates:
𝑃(𝑥) : 𝑥 has a background in python programming,
𝑄(𝑥) : 𝑥 has a background in 𝐶 ++ coding.
5
Then write the statement “There exists a student in this class that has a background in either
python or 𝐶 ++ coding but not both” using predicates.
Solution. The above statement is equivalent to
∃𝑥 : 𝑃(𝑥) ⊻ 𝑄(𝑥).
𝑥 → the number of cars (per hour) entering (exiting) the junction 𝐴 (resp. 𝐵),
𝑦 → the number of cars (per hour) entering Sugar and Jackson roads.
Find the values of 𝑥 and 𝑦, assuming that same number of cars enter and exit a junction.
The numbers (100, 200 and 250) indicate number of cars entering or exiting a junction per
hour.
Solution. We get the following system of equations:
𝑥 + 𝑦 = 200 + 250 = 450,
At A :
At B : 𝑦 + 200 = 𝑥 + 100 ⇔ 𝑥 − 𝑦 = 100.
Solving the system yields 𝑥 = 275, 𝑦 = 175.
6
(24) What is the parity of the permutation 𝜎 = (1, 4, 3, 2) and 𝛿 = (2, 3, 4, 1)?
Solution. We can construct 𝜎 using transpositions as follows:
(1, 2, 3, 4) → (1, 4, 3, 2) = 𝜎
(1, 2, 3, 4) → (2, 1, 3, 4) → (2, 4, 3, 1) → (2, 3, 4, 1) = 𝛿.
Solution. We have
𝐶1→𝐶2+𝐶3 𝑎 + 𝑏 + 𝑐 𝑏 𝑐
𝑎 𝑏 𝑐 1 𝑏 𝑐
𝑐 𝑎 𝑏 −−−−−−−−−→ 𝑎 + 𝑏 + 𝑐 𝑎 𝑏 = (𝑎 + 𝑏 + 𝑐) 1 𝑎 𝑏
𝑏 𝑎 𝑐 𝑎+𝑏+𝑐 𝑎 𝑐 1 𝑎 𝑐
1 𝑏 𝑐
𝑅2→𝑅2−𝑅1 𝑎−𝑏 𝑏−𝑐
−−−−−−−−−→ (𝑎 + 𝑏 + 𝑐) 0 𝑎 − 𝑏 𝑏 − 𝑐 = (𝑎 + 𝑏 + 𝑐) = −(𝑎 + 𝑏 + 𝑐)(𝑎 − 𝑏)(𝑏 − 𝑐).
𝑅3→𝑅3−𝑅1 0 𝑎−𝑏 𝑎−𝑏 0
0
𝑥 + 2𝑦 − 𝑧 = 0
2𝑥 − 𝑦 + 3𝑧 = 0
𝑥+𝑦+𝑧=2
7
Solution. The given system of equations is equivalent to the following matrix equation:
1 2 −1 𝑥 0
2 −1 3 𝑦 = 0 .
1 1 1 𝑧 2
| {z } |{z} |{z}
𝐴 x b
Let us put
0 2 −1 1 0 −1 1 2 0
𝐴1 (b) = 0 −1 3 , 𝐴2 (b) = 2 0 3 , 𝐴3 (b) = 2 −1 0
2 1 1 1 2 1 1 1 2
By calculating the determinants (using cofactor expansions), we find
2. Graph Theory
(30) For the following graphs, list 𝑉 (𝐺), 𝐸 (𝐺) and the values of the incidence mapping 𝜓𝐺 .
8
Solution. For the first graph, 𝑉 (𝐺) = {𝑣 1 , 𝑣 2 , 𝑣 3 }, 𝐸 (𝐺) = {𝑒 1 , 𝑒 2 , 𝑒 3 } and 𝜓𝐺 (𝑒 1 ) =
{𝑣 1 , 𝑣 2 }, 𝜓𝐺 (𝑒 2 ) = {𝑣 1 , 𝑣 3 }, 𝜓𝐺 (𝑒 3 ) = {𝑒 3 }.
For the second graph, 𝑉 (𝐺) = {𝑣 1 , 𝑣 2 , 𝑣 3 , 𝑣 4 }, 𝐸 (𝐺) = {𝑒 1 , 𝑒 2 , 𝑒 3 , 𝑒 4 , 𝑒 5 , 𝑒 6 } and
𝜓𝐺 (𝑒 1 ) = {𝑣 1 , 𝑣 2 }, 𝜓𝐺 (𝑒 2 ) = {𝑣 2 , 𝑣 3 }, 𝜓𝐺 (𝑒 3 ) = {𝑣 2 , 𝑣 3 }, 𝜓𝐺 (𝑒 4 ) = {𝑣 1 , 𝑣 3 }, 𝜓𝐺 (𝑒 5 ) =
{𝑣 3 , 𝑣 4 }, 𝜓𝐺 (𝑒 6 ) = {𝑣 1 , 𝑣 4 }.
(31) Is it possible to construct a graph with 7 vertices of degree 3 and 2 vertices of degree 2?
Solution. Let 𝑉 (𝐺) = {𝑣 1 , 𝑣 2 , 𝑣 3 , 𝑣 4 , 𝑣 5 , 𝑣 6 , 𝑣 7 , 𝑢 1 , 𝑢 2 } where 𝑑 (𝑣 𝑖 ) = 3, 1 ≤ 𝑖 ≤ 7 and
𝑑 (𝑢 1 ) = 𝑑 (𝑢 2 ) = 2. Here 𝑑 (𝑤) denotes the degree of the vertex 𝑤. By the handshaking
lemma, we have
1 ∑︁ 1
|𝐸 (𝐺)| = 𝑑 (𝑤) = (𝑑 (𝑣 1 ) + · · · + 𝑑 (𝑣 7 ) + 𝑑 (𝑢 1 ) + 𝑑 (𝑢 2 ))
2 2
𝑤∈𝑉 (𝐺)
1© ª 1 25
= 3 + · · · + 3 +2 + 2®® = (7 × 3 + 4) = .
2 | {z } 2 2
« 7 times ¬
Since the number of edges |𝐸 (𝐺)| cannot be fraction, the graph with the required properties
in question is not possible.
(32) Let 𝐺 be a graph with 10 vertices such that 𝛿(𝐺) = Δ(𝐺) = 2. How many edges does 𝐺
have?
Solution. Let 𝑢 be any vertex in the graph 𝐺. Then we know (from definition of 𝛿(𝐺) and
Δ(𝐺)) that
1 ∑︁ 1© ª
|𝐸 (𝐺)| = 𝑑 (𝑢) = 2 + 2 + · · · + 2®® = 10.
2 2 | {z }
𝑢∈𝑉 (𝐺)
« 10 times ¬
(33) Let 𝐺 be a graph with Δ(𝐺) = 4. Show that |𝐸 (𝐺)| ≤ 2|𝑉 (𝐺)|.
Solution. Let 𝑢 be any vertex of 𝐺. Then Δ(𝐺) = 4 implies 𝑑 (𝑢) ≤ 4. By the handshaking
lemma, we have
1 ∑︁ 1 ∑︁ ∑︁
|𝐸 (𝐺)| = 𝑑 (𝑢) ≤ 4=2 1 = 2|𝑉 (𝐺)|.
2 2
𝑢∈𝑉 (𝐺) 𝑢∈𝑉 (𝐺) 𝑢∈𝑉 (𝐺)
(34) A graph with 4 edges has a vertex with degree 4, a vertex with degree 1 and one more
vertex. What is the degree of the third vertex?
Solution. For the graph 𝐺, let 𝑉 (𝐺) = {𝑢 1 , 𝑢 2 , 𝑢 3 } with 𝑑 (𝑢 1 ) = 4, 𝑑 (𝑢 2 ) = 1 and
|𝐸 (𝐺)| = 4. By the handshaking lemma, we have
1 ∑︁ 1
|𝐸 (𝐺)| = 𝑑 (𝑢) = (𝑑 (𝑢 1 ) + 𝑑 (𝑢 2 ) + 𝑑 (𝑢 3 ))
2 2
𝑢∈𝑉 (𝐺)
1
⇔4= (4 + 1 + 𝑑 (𝑢 3 )) ⇔ 8 = 5 + 𝑑 (𝑢 3 ) ⇔ 𝑑 (𝑢 3 ) = 3.
2
(35) Find 𝛿(𝐺) and Δ(𝐺) for each of the graphs 𝐺 = 𝐾5 , 𝐾5,7 , 𝑃4 , 𝐶6 . Find |𝑉 (𝐺)| and |𝐸 (𝐺)|
for each of these graphs.
Solution. Recall that 𝐾5 is the unique complete graph with 5 vertices. So, |𝑉 (𝐾5 )| = 5 and
𝐸 (𝐾5 ) = 9. Since 𝐾5 is complete, every vertex in 𝐾5 is connected to the other 4 vertices by
an edge, so 𝛿(𝐾5 ) = Δ(𝐾5 ) = 4.
Recall that 𝐾5,7 is the unique complete bipartite graph with vertex partition 𝑉1 and 𝑉2
(𝑉 (𝐾5,7 ) = 𝑉1 ∪ 𝑉2 ) such that |𝑉1 | = 5 and |𝑉2 | = 7. Also, each vertex in 𝑉1 is connected to
every vertex in 𝑉2 . Thus, |𝑉 (𝐾5,7 )| = 5 + 7 = 12 and |𝐸 (𝐾5,7 )| = 5 × 7 = 35. Next, for any
vertex 𝑢 ∈ 𝑉1 , we have 𝛿(𝑢) = Δ(𝑢) = 7 and for 𝑣 ∈ 𝑉2 , we have 𝛿(𝑣) = Δ(𝑣) = 5.
Recall that 𝑃4 is the path graph with 4 vertices. So, |𝑉 (𝑃4 )| = 4 and |𝐸 (𝑃4 )| = 3. Also,
if 𝑉 (𝑃4 ) = {𝑢 1 , 𝑢 2 , 𝑢 3 , 𝑢 4 } with 𝑢 1 (resp. 𝑢 4 ) be the initial (resp. terminal) vertex of 𝑃4 ,
then 𝑑 (𝑢 1 ) = 𝑑 (𝑢 4 ) = 1 and 𝑑 (𝑢 2 ) = 𝑑 (𝑢 3 ) = 2. Thus, 𝛿(𝑃4 ) = 1 and Δ(𝑃4 ) = 2.
Recall that 𝐶6 is the cycle graph with 6 vertices. Then |𝑉 (𝐶6 )| = 6 and |𝐸 (𝐶6 )| = 6.
Also, we have 𝛿(𝐶6 ) = Δ(𝐶6 ) = 2.
(36) For positive integers 𝑎, 𝑏, show that 𝐾𝑎,𝑏 is regular if and only if 𝑎 = 𝑏.
Solution. Using the ideas from the last problem, we can conclude that if 𝑉1 and 𝑉2 are
vertex partitions (𝑉 (𝐾𝑎,𝑏 ) = 𝑉1 ∪ 𝑉2 ), then |𝑉1 | = 𝑎 and |𝑉2 | = 𝑏. Then for any vertex
𝑢 ∈ 𝑉1 , we have 𝑑 (𝑢) = 𝑏 and for any vertex 𝑣 ∈ 𝑉2 , 𝑑 (𝑣) = 𝑎. Thus, for 𝐾𝑎,𝑏 to be regular
(all vertices have same degree), we must have 𝑑 (𝑢) = 𝑑 (𝑣), that is, 𝑎 = 𝑏. Also, if 𝑎 = 𝑏,
then one can easily check that the graph 𝐾𝑎,𝑎 is regular.
(37) Let 𝐺 be a bipartite graph with bipartition 𝑉 (𝐺) = 𝑉1 ∪ 𝑉2 . If |𝑉1 | = 𝑎 and |𝑉2 | = 𝑎 + 2.
Show that |𝐸 (𝐺)| ≤ 𝑎 2 + 2𝑎.
Solution. Note that 𝐺 may not be a complete bipartite graph. So, for any vertex 𝑢 ∈ 𝑉1 ,
10
we have 𝑑 (𝑢) ≤ 𝑎 + 2 and for any 𝑣 ∈ 𝑉2 , we have 𝑑 (𝑣) = 𝑎. By the handshaking lemma,
we have
!
1 ∑︁ 1 ∑︁ ∑︁
|𝐸 (𝐺)| = 𝑑 (𝑤) = 𝑑 (𝑢) + 𝑑 (𝑣)
2 2 𝑢∈𝑉
𝑤∈𝑉 (𝐺) 1 𝑣∈𝑉2
!
1 ∑︁ ∑︁
≤ (𝑎 + 2) 1+𝑎 1
2 𝑢∈𝑉 𝑢∈𝑉
1 2
1
= ((𝑎 + 2)|𝑉1 | + 𝑎|𝑉2 |)
2
1
= ((𝑎 + 2)𝑎 + 𝑎(𝑎 + 2)) = 𝑎 2 + 2𝑎.
2
(38) Let us the consider the complete bipartite graph with 3 vertices with vertex partition
𝑉 (𝐺) = 𝑉1 ∪𝑉2 with 𝑉1 = {𝑎, 𝑏, 𝑐} and 𝑉2 = {𝑑, 𝑒, 𝑓 }. Find 𝐺 [{𝑎, 𝑏, 𝑑}] and 𝐺 − {𝑑, 𝑒, 𝑓 }.
Solution. Recall that if 𝑉0 ⊂ 𝑉 (𝐺), then 𝐺 [𝑉0 ] is the vertex induced subgraph with
vertex set 𝑉0 . Thus, 𝐺 [{𝑎, 𝑏, 𝑑}] would be
12
On the other hand, 𝐺 − {𝑏, 𝑔} = 𝐺 [𝑉 (𝐺) − {𝑏, 𝑔}] = 𝐺 [{𝑎, 𝑐, 𝑑, 𝑒, 𝑓 , ℎ}] is the vertex
induced subgraph shown below:
Solution. Recall that the adjacency matrix of a graph 𝐺 is a square matrix of size
|𝑉 (𝐺)| × |𝑉 (𝐺)|. In this case, |𝑉 (𝐺)| = 4. If 𝑢 𝑎 and 𝑢 𝑏 are any two vertices of 𝐺. Then
(
#{edges of 𝐺 between 𝑢 𝑎 and 𝑢 𝑏 }, 𝑎 ≠ 𝑏,
[ 𝐴(𝐺)] 𝑎𝑏 =
2#{loops on 𝑢 𝑎 }, 𝑎 = 𝑏.
Thus
0 1 0 0
1 0 1 1
𝐴(𝐺) =
0 1 0 0
0 1 0 2
(42) Find the adjacency matrix of 𝐾4 .
Solution. Recall that 𝐾4 is the unique complete (simple) graph with 4 vertices. Thus, we
have
0 1 1 1
1 0 1 1
𝐴(𝐾4 ) =
1 1 0 1
1 1 1 0
13
(43) Note from Problem 42 that the sum of the entries Í inÍ𝐴(𝐾4 ) equals 12 which equals 2|𝐸 (𝐾4 )|.
Show that for any graph 𝐺 with 𝑛 vertices, 𝑛𝑎=1 𝑛𝑏=1 [ 𝐴(𝐺)] 𝑎𝑏 = 2|𝐸 (𝐺)|.
Solution. Let 𝑉 (𝐺) = {𝑢 1 , 𝑢 2 , · · · , 𝑢 𝑛 }. From handshaking lemma, we have
∑︁𝑛
𝑑 (𝑢 𝑎 ) = 2|𝐸 (𝐺)|. (2.1)
𝑎=1
Using the properties of row sums of 𝐴(𝐺), we find
𝑛
∑︁
[ 𝐴(𝐺)] 𝑎𝑏 = 𝑑 (𝑢 𝑎 ). (2.2)
𝑏=1
Using (2.2) in (2.1) yields
𝑛 ∑︁
∑︁ 𝑛
[ 𝐴(𝐺)] 𝑎𝑏 = 2|𝐸 (𝐺)|.
𝑎=1 𝑏=1
14